Equivalencia de One Flux Quantum y Zero Flux

En la revisión de Ady Stern del efecto Quantum Hall, dice de un sistema cuántico hall "El espectro en Φ = Φ 0 es el mismo que el espectro en Φ = 0 ..." ¿Alguien puede explicar por qué sucede esto? Parece que el campo magnético aplicado ciertamente cambia el hamiltoniano y, por lo tanto, el espectro, pero aparentemente no cuando el flujo es un cuanto único.

También pido disculpas por cualquier error de novato o si esto se responde en otro lugar, soy bastante nuevo en stackexchange. Gracias.

¿Qué reseña? este ?
si ese es el

Respuestas (2)

Los estados en Φ y Φ + Φ 0 están relacionados por una transformación de calibre [1] y, por lo tanto, el espectro debe ser el mismo. Para concretar, hablemos de electrones fijados a un anillo de radio. R . Parametrizar la función de onda ψ por la longitud del arco yo . La periodicidad requiere que

ψ ( yo + 2 π r ) = ψ ( yo ) .
Hay algo de hamiltoniano H en el anillo - H debe ser invariante de calibre, por lo que es una función solo de la posición yo y la derivada invariante de guage D = i yo + A , dónde A es el calibre.

Ahora tomemos la transformación de calibre A A + norte / r , ψ mi i norte yo / r ψ . Esto respeta el límite periódico iff norte es un número entero, por lo que toma estados propios del hamiltoniano a estados propios del hamiltoniano. Ahora usando la relación

A d yo = S × A d a = S B d a Φ ,
vemos que nuestra transformación de calibre es equivalente a cambiar el cuanto de flujo por Φ 0 .

Tenga en cuenta que también podríamos usar una transformación de prueba de calibre de la forma A A + norte + θ / 2 π r . Podemos hacer esto siempre y cuando cambiemos nuestra condición bondary a

ψ ( yo + 2 π r ) = mi i θ ψ ( yo ) .
En particular, podemos transformar el campo de calibre a cero y quedarnos sin A pero con una condición de contorno no trivial arriba. Entonces, si lo desea, puede olvidarse del campo magnético y el campo de calibre y pensar en cambio en un problema sin carga pero con la condición límite "retorcida". Esto debería tener sentido - no hay magnetismo en d = 1 así que debería poder deshacerme básicamente de A . La condición de contorno es el único vestigio que queda.

[1] Lo llamo transformación de calibre, pero en el caso físico de un anillo enhebrado por flujo, esta transformación de calibre no se puede extender a todo el espacio. Esto se deriva del hecho de que cambia la cantidad de flujo que pasa por el anillo, pero eso es invariante en el calibre. Sin embargo, el espectro electrónico no sabe que esta transformación no se puede extender a todo el espacio, por lo que todavía funciona.

Esto se debe a la hipótesis adiabática: si el procedimiento de insertar un cuanto de flujo en el sistema es adiabático y el sistema está en un estado propio del hamiltoniano ψ norte con energía propia mi norte luego, permanecerá bloqueado en este estado propio (hasta un factor de fase global) durante y al final del proceso.

La declaración es sobre el espectro, no sobre el estado.